53-я Международная Математическая Oлимпиада
Аргентина, Мар-дель-Плата, 2012 год


Дано целое число $n\ge 3$ и действительные положительные числа ${{a}_{2}}$, ${{a}_{3}}$, $\ldots $, ${{a}_{n}}$ такие, что ${{a}_{2}}{{a}_{3}}\ldots {{a}_{n}}=1$. Докажите, что $${{(1+{{a}_{2}})}^{2}}{{(1+{{a}_{3}})}^{3}}\ldots {{(1+{{a}_{n}})}^{n}} > {{n}^{n}}.$$
посмотреть в олимпиаде

Комментарий/решение:

  3
2019-04-16 22:30:00.0 #

$$x_k=(1+a_k)^k, \qquad k=2,3,...,n$$

$$x_k=(1+a_k)^k=(\frac{1}{k-1}\cdot (k-1)+a_k)^k=$$ $$=\Bigg(\frac{\underbrace{1+1+...+1}_{k-1}}{k-1}+a_k\Bigg)^k\geq \Bigg(k\sqrt[k]{\frac{1}{(k-1)^{k-1}}a_k}\Bigg)^k=k^k\frac{1}{(k-1)^{k-1}}a_k$$

$$ \prod_{k=2}^n x_k \geq \prod_{k=2}^n\Bigg(k^k\frac{1}{(k-1)^{k-1}}a_k\Bigg)=\frac{ \prod_{k=2}^nk^k}{\prod_{k=2}^n(k-1)^{k-1}}\prod_{k=2}^na_k=n^n$$

Причем равенство достигается в том и только в том случае, когда $a_k=\frac{1}{k-1}.$ По условию задачи известно, что произведение чисел $a_2,a_3,...,a_n$ равно единице:

$$ a_2 \cdot a_3 \cdot ... \cdot a_{n-1} \cdot a_n=\frac{1}{1 \cdot 2 \cdot ... \cdot (n-1)}=1 \Leftrightarrow (n-1)!=1 \Rightarrow n=2$$

Равенство достигается только тогда, когда $n=2$.

$$ \Rightarrow \quad \forall n\geq 3: \qquad (1+a_2)^2(1+a_3)^3...(1+a_n)^n>n^n$$